domanda sui ciclici ma non prprio....

Messaggioda sastra81 » 15/01/2007, 19:40

Supponiamo di avere il <x>(sottogruppo generato da x ove x è un elemento di un gruppo G)ciclico infinito
supponiamo di avere <x^p^n> e <x^q^m> sottogruppi normali di G allora devo dimostare che <x>=<x^p^n,x^q^m>
Una idea iniziale è la seguente poiche <x^p^n> e <x^q^m> sottogruppi normali di G allora posso considerare i seguenti qozienti <x>/<x^p^n> (che ha ordina p^n GIUSTO QUI HO UN DUBBIO)
<x>/ <x^q^m>(che ha ordine q^m) allora a questo punto che faccio applico il teorema di Lagrange?
AIUTOOOO
GRAZIE
SASTRA
sastra81
Junior Member
Junior Member
 
Messaggio: 103 di 216
Iscritto il: 26/10/2005, 17:13

Messaggioda Ravok » 16/01/2007, 08:52

Ciao..
per quanto riguarda $<x>$$/<x^(p^n)>$ l'ordine direi che è $p^n$, stessa cosa per quanto riguarda $q^m$... fino a qui non c'è problema... per il teorema di Lagrange, ti risulterebbe una uguaglianza di ordini....io non capisco in verità cosa significa$<x>$$=<x^(p^n),x^(q^m)>$....vorresti dire che
$<x>$$={x^i, $tc $ i in ZZ} =$$<x^(p^n),x^(q^m)>$...non capisco il significato di quella virgola...
ciao
Non si può essere entrambe le cose...(ik)
Ravok
Junior Member
Junior Member
 
Messaggio: 41 di 171
Iscritto il: 29/09/2006, 16:49


Torna a Algebra, logica, teoria dei numeri e matematica discreta

Chi c’è in linea

Visitano il forum: Nessuno e 1 ospite